Đến nội dung

dinhthanhhung nội dung

Có 108 mục bởi dinhthanhhung (Tìm giới hạn từ 29-04-2020)



Sắp theo                Sắp xếp  

#513551 Tìm số số tốt

Đã gửi bởi dinhthanhhung on 17-07-2014 - 23:48 trong Số học

Số tốt là một số mà có thể viết dưới dạng tổng các chữ số của một số chính phương . Ví dụ : 13 là số tốt vì 7^2=49 , 4+9=13 . Tìm số số tốt trong 1,2,3,...,2007 ?




#512186 Trung bình cộng của các số có 9 chữ số , trong đó có 5 chữ số 5 v...

Đã gửi bởi dinhthanhhung on 10-07-2014 - 22:41 trong Tổ hợp - Xác suất và thống kê - Số phức

Tìm trung bình cộng của các số có 9 chữ số , trong đó có 5 chữ số 5 và 4 chữ số 4 ?




#509842 Tồn tại số này là bội số kia

Đã gửi bởi dinhthanhhung on 29-06-2014 - 17:35 trong Số học

Với $p$ là số nguyên tố lớn hơn $5$ , xét tập $A=\left \{ p-n^2|n\in \mathbb{N},n^2<p \right \}$ .

Chứng minh trong $A$ có 2 số khác $1$ mà số này là bội số kia .




#509298 Bài toán trò chơi bốc vật

Đã gửi bởi dinhthanhhung on 27-06-2014 - 00:02 trong Tổ hợp và rời rạc

Có $2018$ viên kẹo. Hai người thay phiên nhau bốc kẹo, biết rằng mỗi lần chỉ được bốc $3$,$4$ hoặc $7$ viên kẹo; nếu còn lại $1$ hoặc $2$ viên thì được bốc hết. Người bốc viên kẹo cuối cùng là người chiến thẳng. Hỏi ai là người có chiến thuật thắng, người đi trước hay người đi sau?

 

Cách làm không hay nhưng hiệu quả :))

Ta xem xét trường hợp nhỏ để tìm quy luật . n=1,2,3,4 người một thắng . n=5,6,7 người hai thắng . n=8,9,10,11,12,13,14 người một thắng . n=15,16,17 người hai thắng . Từ đây thấy sau 3 trường hợp người một thua và trường hợp thua ở cuối ngay sau 7 trường hợp thắng thì người một thắng tiếp 7 trường hợp sau ( giải thích : , trường hợp đầu bốc 3 , trong 3 trường hợp tiếp chọn 4 viên , 3 trường hợp sau chọn 7 viên thì sẽ đưa về 3 trường hợp mà người đầu bốc thua , hay người hai sẽ thua ) . Và ta cũng thấy sau 7 trường hợp người một thắng thì có 3 trường hợp người hai thắng , do người một bốc như thế nào cũng đưa về trường hợp người đầu thắng hay người hai thắng . 

Đến đây mọi chuyện đã sáng tỏ . Ta có điều sau : với n=0,1,2,3,4,8,9 mod 10 thì người một thắng , trường hợp còn lại người hai thắng .

2018=8 mod 10 vậy người một thắng .




#508577 Topic về tổ hợp, các bài toán về tổ hợp

Đã gửi bởi dinhthanhhung on 23-06-2014 - 14:14 trong Tổ hợp và rời rạc

 

Bài 39: Đặt $21$ điểm trên đường tròn. Chứng minh rằng: có ít nhất $100$ cặp điểm tạo ra cung nhỏ hơn hoặc bằng $120^0$

 

Xét graph đỉnh là $21$ điểm trên, $2$ đỉnh nối với nhau nếu cung tạo bởi chúng lớn hơn $120^0$ .

Dễ thấy đây là graph 3-free nên số cạnh tối đa là $\left \lfloor \frac{21^2}{4} \right \rfloor=110$

Do đó số cặp đỉnh tạo cung nhỏ hơn hoặc bằng $120^0$ là $C_{21}^{2}-110=100$




#495533 Tìm số nguyên dương $k$ nhỏ nhất sao cho trong $k$ phần t...

Đã gửi bởi dinhthanhhung on 27-04-2014 - 19:53 trong Tổ hợp và rời rạc

Tìm số nguyên dương $k$ nhỏ nhất sao cho trong $k$ phần từ tùy ý của tập ${1;2;...;49;50}$ luôn chứa $3$ số là độ dài $3$ cạnh của $1$ tam giác vuông 

 

Lời giải:

Xét tập A={1,2,3,5,8,13,21,34} gồm 8 phần tử và không có 3 số nào là độ dài 3 cạnh tam giác .

Do đó k>8 .

Ta chứng minh k=9 thỏa mãn .

Phản chứng có 1 tập 9 phần tử không thỏa mãn là B={a_1,...,a_9}

Dễ chứng minh 50>=2F_8+F_7>=55 (MT)

Do đó có dpcm

 

* Nhận xét : bài trên là một trong những áp dụng của dãy Fibonacci .




#493609 Hỏi sau hữu hạn các bước có đưa bảng về toàn số 0 hay không ?

Đã gửi bởi dinhthanhhung on 17-04-2014 - 21:40 trong Tổ hợp và rời rạc

Bài này đề đúng là thực hiện các bước a,b so le nhau . Nếu không thì sẽ rất lộn xộn (!)

Lời giải :

Ta thấy sau khi nhân đôi một hàng nào đó , rồi trừ một cột nào đó thì trên bảng sẽ có ít nhất một số lẻ .

Do vậy bước cuối cùng là bước nhân đôi các số trong một hàng .

Mặt khác sau khi nhân , ta được bảng toàn số 0 , vậy bước đó không thay đổi gì cả .

Điều này đưa ta tới kết luận là bảng ban đầu phải có số 0 , điều này vô lí với giả thiết .

Vậy không tồn tại hữu hạn buớc .

 

Xin lỗi vì đã chém đề =)) Hôm nay nghĩ lại thì với điều kiện như bạn đã cho , ta có lời giải như sau :

Xét một cột và các ô chứa số 1 nằm trên hàng đó ta nhân hai các hàng tương ứng sau đó trừ 1 cột đó đi ( đến khi xuất hiện số 1 thì thôi ) , nếu không có số 1 nào thì ta trừ 1 . Dễ thấy tổng của tất cả các số sẽ giảm sau các biến đổi , đồng thời các biến đổi đảm bảo không có số nào thành số 0 . Do đó đến một lúc nào đấy cột đó sẽ toàn số 1 và lúc này trừ 1 đi ta được toàn số 0.

Thực hiện lần lượt trên các cột , có được bảng toàn số 0.




#488172 CMR: Sau một số bước có thể bỏ hết các viên bi vào hộp.

Đã gửi bởi dinhthanhhung on 21-03-2014 - 21:47 trong Tổ hợp và rời rạc

Cho m viên bi đựng trong n cái hộp (m, n>4), có thể có hộp không có bị và số bị trong hộp không nhất thiết bằng nhau. Ta thực hiện một trò chơi như sau: Mỗi lần lấy ra 2 viên bi ở hai hộp nào đó và bỏ vào hộp thứ 3. CMR: Sau một số bước có thể bỏ hết các viên bi vào một hộp.

 

Bài này chắc quy nạp là hướng nghĩ dễ nhất rồi .

 

Lời giải :

Ta kí hiệu (a,b,c,d) là 4 hộp mà hộp 1 có a kẹo , hộp 2 có b kẹo , ...

*m=4 thì có tối đa 4 hộp đựng kẹo là (1,1,1,1),(1,1,2,0),(1,3,0,0),(2,2,0,0)

Ta biến đổi như sau : (1,1,1,1)->(3,1,0,0)->(2,0,2,0)->(2,1,0,1)->(4,0,0,0)

Do đó m=4 là OK .

*Giả sử số kẹo là m thì OK , ta xét trường hợp m+1 kẹo .

Đánh dấu 1 viên kẹo . Theo giả thiết quy nạp thì chuyển được m kẹo còn lại vào 1 hộp .

Nếu hộp đó chứa viên đánh dấu thì xong luôn , ta xét nó không chứa viên đó .

Biến đổi như sau : (1,m,0,0)->(0,m-1,2,0)->(0,m-2,1,2)->(2,m-3,0,2)->(1,m-1,0,1)->(0,m+1,0,0) 

*Kết luận: ...

 

p/s :

 

Đề là ''bi'' mà bạn

 

 

Ừ =))! Mình không đọc kỹ . 




#486305 Chứng minh rằng tồn tại ít nhất một phần tử của tập hợp A và một phần tử của...

Đã gửi bởi dinhthanhhung on 08-03-2014 - 20:03 trong Tổ hợp và rời rạc

Phản chứng không có 2 phần tử nào như vậy . 

Theo nguyên lí Dirichlet tồn tại 1 tập có số phần tử lớn hơn hoặc bằng 1005 , ta giả sử là tập A và |A| =1005+x (x tự nhiên)

Từ điều phản chứng thì có ít nhất 1005+x số không thuộc B (lấy 2008 trừ đi từng phần tử của A). Vậy |B|<=1003-x

Suy ra |A|+|B|<=2008 (vô lí)




#485579 Chứng minh rằng: số dấu cộng trên bảng trên luôn không nhỏ hơn $n$...

Đã gửi bởi dinhthanhhung on 02-03-2014 - 23:41 trong Tổ hợp và rời rạc

 

Cho một hình vuông $n \times n$ với $n \geq 4$. Ta viết $n$ dấu cộng vào các ô trên một đường chéo lớn của hình vuông trên, các ô còn lại được diền vào dấu trừ. Ta có thể thay đổi bảng trên bằng cách đổi dấu trong các ô cùng một hàng hoặc một cột. Chứng minh rằng: số dấu cộng trên bảng trên luôn không nhỏ hơn $n$ sau một số hữu hạn lần thay đổi.

 

Lời giải :

 

Gọi x_i là số lần biến đổi ở cột thứ i , y_i là số lần biến đổi ở hàng thứ i .

Do đó số lần biến đổi ở ô cột a , hàng b là x_a+y_b . Nếu giá trị này chẵn thì dấu ban đầu được bảo toàn , nếu lẻ thì dấu ban đầu đổi ngược .

Giờ ta phản chứng một lúc nào đó trên bảng có ít hơn n ô được đánh dấu cộng . Theo nguyên lí Dirichlet tồn tại một cột không có số nào được đánh dấu cộng . KMTQ ta giả sử đó là cột 1 .

Khi đó x_1+y_1 phải lẻ , x_1+y_i (i từ 2->n) phải chẵn . Do đó y_i cùng dấu (i từ 2->n) và tất cả khác dấu y_1.

Ta xét các cột còn lại , theo tính chẵn lẻ thì mỗi cột sẽ có 2 hoặc n-2 ô được đánh dấu cộng (dễ chứng minh) , mà n lớn hơn 3 nên mỗi cột có ít nhất 2 .

Do đó số ô được đánh dấu cộng ít nhất là 2(n-1) lớn hơn n-1 .

Vậy phản chứng sai , có dpcm !

 

P/s : Phần mềm gõ công thức bị lỗi , híc :|




#483016 Hỏi sau hữu hạn các bước có đưa bảng về toàn số 0 hay không ?

Đã gửi bởi dinhthanhhung on 13-02-2014 - 23:32 trong Tổ hợp và rời rạc

Bài này đề đúng là thực hiện các bước a,b so le nhau . Nếu không thì sẽ rất lộn xộn (!)

Lời giải :

Ta thấy sau khi nhân đôi một hàng nào đó , rồi trừ một cột nào đó thì trên bảng sẽ có ít nhất một số lẻ .

Do vậy bước cuối cùng là bước nhân đôi các số trong một hàng .

Mặt khác sau khi nhân , ta được bảng toàn số 0 , vậy bước đó không thay đổi gì cả .

Điều này đưa ta tới kết luận là bảng ban đầu phải có số 0 , điều này vô lí với giả thiết .

Vậy không tồn tại hữu hạn buớc .




#482275 AC vuông góc với CE

Đã gửi bởi dinhthanhhung on 09-02-2014 - 19:21 trong Hình học

cho đường tròn (O) cát tuyến PAB vẽ tiếp tuyến PC kẻ đường kính CD (CD cắt AB) OP cắt BD tại E. Chứng minh AC vuông góc với CE

 

Lấy F là trung điểm AB

Tg FAC ~ Tg ODE -> Tg BAC ~ Tg CDE -> dpcm .




#481960 Topic về tổ hợp, các bài toán về tổ hợp

Đã gửi bởi dinhthanhhung on 08-02-2014 - 16:01 trong Tổ hợp và rời rạc

Bài 29:Trong một phòng thi gồn có 9 thí sinh được xếp ngồi xung quanh một bàn tròn.TRong ngân hàng đề có 9 loại đề khác nhau mỗi loại có nhiều bản.Một cách phát đề được coi là hợp lệ nếu mỗi thí sinh được nhận chỉ một đề và hai thí sinh ngồi cạnh nhau thì nhận được hai loại đề khác nhau.Hỏi có tất cả bao nhiêu cách phát đề hợp lý?

 

Bài này giải bằng truy hồi . Cách giải ở đây: http://diendantoanho...òn/#entry479746

p/s: Mãi mà không nghĩ ra ...




#481924 CMR: $KI\perp MN$

Đã gửi bởi dinhthanhhung on 08-02-2014 - 13:45 trong Hình học

Cho tam giác ABC, trên các phần kéo dài AB, CB về phía B lấy M, N sao cho AM=CN=p (nửa chu vi tam giác). Gọi I là tâm nội tiếp tam giác ABC, O là tâm ngoại tiếp, kẻ đường kính BK của (O). CMR: $KI\perp MN$

* Ở dưới các điểm mình đặt tên các điểm khác đề bài .

Lời giải :

Giả sử $NI$ cắt $(O)$ tại $P$ . Vẽ phân giác $PM$ của $\widehat{BPC }$, cắt $BC$ tại $D$ .

Có $IM^2=CM^2=MD.MA$ nên $IM$ là tiếp tuyến của $(AID)$ .

Do đó $\widehat{IDA}=\widehat{AIP}=\widehat{PAX}=\widehat{PAQ}$ , suy ra $ID//QM$ hay $ID\perp BC$ 

Vậy :$\frac{PB}{PC}=\frac{DB}{DC}=\frac{AJ}{AK}$ kết hợp với $\widehat{BPC}=\widehat{BAC}=\widehat{JAK}$ có $\Delta PBC ~ \Delta AJK$

Suy ra :$\widehat{AJK}=\widehat{PBC}=\widehat{PAC}$ , được $AP//JK$ , dễ thấy ngay :$NI\perp JK$ (dpcm).

 

 

Hình gửi kèm

  • Capture.PNG



#481826 Trận 3 - Tổ hợp rời rạc

Đã gửi bởi dinhthanhhung on 07-02-2014 - 23:25 trong Thi giải toán Marathon Chuyên toán 2014

Quy ước : đường xuất phát từ thành phố X đến thành phố Y gọi là đường đi đối với thành phố X và là đường kết đối với thành phố Y ( X đi , Y kết ) .

 

Lời giải :

Ta xét thành phố H có nhiều đường đi nhất ( giả sử có $x$ đường đi và $y$ đường kết thì có $x$ thành phố kết , $y$ thành phố đi) .

Theo giả thiết thì trong hai thành phố bất kỳ mà H đi  ( hoặc kết ) thì không có con đường nào nối .

Chọn một thành phố mà H đi và một thành phố mà H kết thì giữa chúng có thể có ( một và chỉ một ) đường nối nên số đường nhiều nhất tạo được là $xy$ .

Bây giờ ta xét một trong $z$ thành phố mà không có đường đi và kết H thì có thể xây tối đa $x+y$ đường nối nên số đường nhiều nhất là $z(x+y)$

Tổng số đường xây nhiều nhất là $A=x+y+xy+z(x+y)$ và thêm đó $x+y+z+1=210$

Theo BDT quen thuộc : $A=xy+y(z+1)+(z+1)x\leq \frac{(x+y+z+1)^2}{3}=\frac{210^3}{3}$

Dấu bằng xảy ra với cách xây như sau : có 3 nhóm thành phố A,B,C , mỗi nhóm 70 . Mối thành phố ở nhóm A có đường đi tới thành phố nhóm B , mối thành phố ở nhóm B có đường đi tới mỗi thành phố nhóm C và mối thành phố ở nhóm C có đường đi tới thành phố nhóm A .




#480508 Loại bất cứ số nào ra khỏi tập hợp đi thì tập hợp $2013$ số còn lại...

Đã gửi bởi dinhthanhhung on 02-02-2014 - 21:07 trong Tổ hợp và rời rạc

Tồi tại hay không một tập hợp gồm $2014$ số nguyên dương với tính chất: loại bất cứ số nào ra khỏi tập hợp đi thì tập hợp $2013$ số còn lại có thể chia thành $2$ tập con với tổng các số (thuộc mỗi tập con đó) là bằng nhau

 

Không tồn tại !

 

Đầu tiên , ta chứng minh rằng nếu có 1 tập như vậy thì tất cả các số trong tập phải bằng nhau .

Ta xét tập nhỏ nhất thoả mãn điều kiện trên và tất cả không cùng bằng nhau !

Dễ thấy bỏ đi 1 số bất kì trong tập thì tổng các số còn lại chia hết cho 2 nên số bỏ đi phải đồng dư với tổng các số trong tập với modulo 2

Tương tự như vậy ta có nhận xét quan trọng là các số trong tập phải cùng tính chẵn lẻ .

Trường hợp 1 , các số đều chẵn , ta xét tập gồm các số là thương của các số trong tập trên chia cho 2 . Dễ thấy tập này thoả mãn nhưng lại nhỏ hơn tập ban đầu -> vô lí !

Trường hợp 2 , các số đều lẻ , ta xét tập gồm các số là thương của các số trên sau khi cộng thêm 1 rồi chia cho 2 . Tập này nhỏ hơn và thoả mãn -> vô lí !

Vậy nhận xét được chứng minh .

Sau cùng ta thấy 2013 số bằng nhau không chia được thành 2 tập tổng các số bằng nhau .




#480497 $f:N\rightarrow N$ toàn ánh.$\frac{f(n)}...

Đã gửi bởi dinhthanhhung on 02-02-2014 - 20:54 trong Dãy số - Giới hạn

Bài này kì quá.

 

Nếu $f$ bậc 1 thì $a$ hằng số dương.

Nếu $f$ bậc lớn hơn 1 thì dương vô cùng.

 

f có toàn ánh không bạn ? 




#478673 Cho một dãy số tăng 1,3,4,9,10,12,13,... chứa tất cả số nguyên dương là lũy t...

Đã gửi bởi dinhthanhhung on 23-01-2014 - 21:31 trong Tổ hợp và rời rạc

Cho một dãy số tăng 1,3,4,9,10,12,13,... chứa tất cả số nguyên dương là lũy thừa của 3 hoặc là tổng của các lũy thừa của 3 . 

Tính số hạng thứ 1998 ?




#476762 Trận 1 - Số học

Đã gửi bởi dinhthanhhung on 11-01-2014 - 22:42 trong Thi giải toán Marathon Chuyên toán 2014

Mở rộng bài toán :

 

Tìm các số nguyên không âm $x,y$ thỏa mãn :

$x^n=y^n+\sqrt[k]{y+1}$ trong đó $n$ thuộc $\mathbb{N}^*,n>1$ , $0\leq k\leq 1$

Hướng giải : Ta chứng minh được $y^n<x^n\leq (y+1)^n$ 

 

$k \ge 2$ thì đúng hơn




#476753 Trận 1 - Số học

Đã gửi bởi dinhthanhhung on 11-01-2014 - 22:29 trong Thi giải toán Marathon Chuyên toán 2014

Lời giải :

 

Do $y$ là số nguyên không âm nên : $y^2+\sqrt{y+1}\leq (y+1)^2\Leftrightarrow 4y^2+2y\geq 0$ (*) đúng 

Vì vậy : $x^2\leq (y+1)^2$

Dễ thấy $x^2> y^2$

Kết hợp hai điều trên , ta có $x^2=(y+1)^2$ ; đồng thời (*) xảy ra dấu bằng nên $y=0$ , từ đó $x=1$

Kết luận : $(x,y)=(1,0)$

 

Tại sao "dễ thấy"? Hãy chứng minh. Trừ 1đ.

Không thử lại: trừ 1đ.

$d=8$

$d_{mr}=4;d_{tl}=0;d_{t}=0$

$S=44$




#476355 Chứng minh có 1 số xuất hiện ít nhất n lần.

Đã gửi bởi dinhthanhhung on 09-01-2014 - 19:29 trong Tổ hợp và rời rạc

Cho một bảng ô vuông $n\times n$. Điền mỗi ô vuông của bảng bằng một số nguyên sao cho 2 ô nằm cạnh nhau hơn kém nhau không quá 1 đơn vị. Chứng minh có 1 số xuất hiện ít nhất n lần.

 

Đầu tiên có nhận xét sau : Cho một số số được viết liên tiếp nhau sao cho khoảng cách giữa hai số liên tiếp không quá 1 . Trong dãy đó , nếu tồn tại một số nhỏ hơn (hoặc bằng) $k$ và một số lớn hơn (hoặc bằng) $k$ thì sẽ có ít nhất một số trong dãy là $k$ .

 

Trở lại với bài toán :

Gọi $m$ là số lớn nhất trong $n$ số nhỏ nhất của mỗi hàng .

Ta chứng minh $m$ là số nhỏ nhất .

Thật vậy , nếu mỗi hàng đều có một số lớn hơn hoặc bằng $m$ ta áp dụng nhận xét cho từng hàng (đều có số nhỏ hơn hoặc bằng $m$ là số nhỏ nhất mỗi hàng ) thì mỗi hàng có ít nhất một số là $m$

Xét trường hợp tồn tại một hàng nào đó chỉ có toàn số nhỏ hơn $m$ . Ta lại có hàng chứa số $m$ là hàng gồm toàn số lớn hơn hoặc bằng $m$ . Áp dụng nhận xét cho mỗi cột thì mỗi cột có ít nhất một số là $m$

Từ trên có dpcm




#475358 $f:N\rightarrow N$ toàn ánh.$\frac{f(n)}...

Đã gửi bởi dinhthanhhung on 04-01-2014 - 22:19 trong Dãy số - Giới hạn

Cho $f:N\rightarrow N$ và $f$ toàn ánh . 

Giả sử tồn tại $\frac{f(n)}{n}\rightarrow a$ khi $n\rightarrow +\infty$

Tìm $a$ ? 




#474432 Xét đa thức bậc hai $f(x)=ax^{2}+bx+c$(a khác 0).Biết...

Đã gửi bởi dinhthanhhung on 01-01-2014 - 13:00 trong Đa thức

 

2/ Cho $f(x)=x^{4}-3x^{3}+(2+a)x^{2}-x+a$. Cmr với mọi số nguyên $a$ thì $f(x)$ không có nghiệm nguyên lẻ.

3/ Cho đa thức f(x) xác định với mọi giá trị của $x$ nguyên dương và tm:
$\left\{\begin{matrix}f(1)=25 & & \\ f(1)+f(2)+...+f(n)=n^{2}f(n)(n\in \mathbb{N}*) & & \end{matrix}\right.$

Tính $f(2014)$

 

 

Bài 2 :

Khi x lẻ , dù a chẵn hay lẻ ta đều có f(x) lẻ do đó khác 0 

 

Bài 3 :

Ta có : $f(n)=n^2f(n)-(n-1)^2f(n-1)$ hay $f(n)=\frac{(n-1)}{n+1}f(n-1)$ 

Từ trên tính được $f(n)$ theo $n$ và $f(1)$ : $f(n)=\frac{2}{(n+1)n}f(1)$




#473807 CM dãy số không tuần hoàn

Đã gửi bởi dinhthanhhung on 29-12-2013 - 20:31 trong Dãy số - Giới hạn

Cho dãy số thực $\left \{ x_n \right \}_{ n=1}^{\infty}$ thoả mãn : 

$x_{2013n+31}=12,x_{2013n+12}=31$ với mọi $n\geq 0$

$x_{n}=x_{2013n}$ với mọi $n\geq 1$

CM dãy số không tuần hoàn 

( Dãy số tuần hoàn khi $\exists m,T\in \mathbb{N}^*:x_n=x_{n+T}$ với mọi $n\geq m$ )




#473555 Đề thi Chọn đội tuyển KHTN (vòng 1)

Đã gửi bởi dinhthanhhung on 28-12-2013 - 22:34 trong Thi HSG cấp Tỉnh, Thành phố. Olympic 30-4. Đề thi và kiểm tra đội tuyển các cấp.

Bài 3 vòng 1 :

Để cho tiện , em kí hiệu $\overrightarrow{GA}=vGA$

Ta có :$vGA1=\frac{p-c}{a}vGB+\frac{p-a}{c}vGC$ và $vGA2=\frac{p-b}{a}vGB+\frac{p-a}{c}vGC$

Tương tự rồi cộng lại được : $3(vGG1+vGG2)=(vGA1+vGB1+vGC1)+(vGA2+vGB2+vGC2)=2(vGA+vGB+vGC)=v0$